Intégrale hivernale

Bonjour
De quoi vous réchauffer dans ce froid glacial $$

\int_0^1 \ln(1-\sqrt x)\ln^2(1+\sqrt x)=\frac{11}{4}-\frac{\pi^2}{3}.

$$ C'est simple mais ardu (sans les $\zeta$ de FDP)
Je suis très confiant que cette intégrale va résister jusqu’à l'hiver prochain.

FDP est occupé par ces intégrales automnales.
YvesM ne s’intéresse plus à ces bestioles .
Le 😄 Farceur


Réponses

  • Gebrane:

    M'étonnerait qu'elle tienne une seule semaine. B-)-
  • Si quelqu'un peut faire sauter l’intégrale en moins d'une semaine, qu'on m'interdise l’accès au site pendant une année.
    Je suis confiant.
    Le 😄 Farceur


  • Gebrane:

    Ne donne pas le bâton pour te battre.
  • Il faut trouver d'abord le bâton= la solution.
    Je prends le risque, je suis confiant.
    Le 😄 Farceur


  • Bonjour,

    L’intégrande admet une primitive explicite. En trichant avec un programme, on trouve cette primitive et la démonstration prend moins d’une demi-heure de calcul d’une dérivée.

    Limites-tu les méthodes à utiliser (résidus, dérivée sous le signe somme) ou pas ?
  • Ah non YvesM est la.92702
    Le 😄 Farceur


  • (oui , j'autorise tout sauf de partir par une primitive suggérée par un logiciel ) voici le début de la solution que j'ai (c'est très compliqué la suite )
    $$\begin{align} \int_0^1 \ln(1-\sqrt x)\ln^2(1+\sqrt x) &=\int_0^1 2x\ln(1-x)\ln^2(1+x)dx\\ &=\int_0^1 4x\ln(1-x)\sum_{n=1}^\infty \frac{(-1)^{n+1}H_n x^{n+1}}{n+1}dx\\ &=4\sum_{n=1}^\infty \frac{(-1)^{n+1}H_n}{n+1}\int_0^1 x^{n+2}\ln(1-x)dx\\ &=4\sum_{n=1}^\infty \frac{(-1)^{n}H_n}{n+1}\frac{H_{n+3}}{n+3}\\ &=4\sum_{n=1}^\infty \frac{(-1)^{n}H_nH_{n+3}}{(n+1)(n+3)} \end{align}$$
    Le 😄 Farceur


  • C'est un peu long mais j'ai détaillé quelques points.

    \begin{align}J&=\int_0^1 \ln^2(1+\sqrt{x})\ln(1-\sqrt{x})\,dx\\
    K&=\int_0^1 \ln^3\left(\frac{1-\sqrt{x}}{1+\sqrt{x}}\right)\,dx\\
    &\overset{y=\frac{1-\sqrt{x}}{1+\sqrt{x}}}=4\int_0^1 \frac{(1-x)\ln^3 x}{(1+x)^3}\,dx\\
    &\overset{IPP}=-12\int_0^1 \frac{\ln^2 x}{(1+x)^2}\,dx\\
    &\overset{IPP}=-12\left[\left(1-\frac{1}{1+x}\right)\ln^2 x\right]_0^1 +24\int_0^1 \frac{\ln x}{1+x}\,dx\\
    &=24\int_0^1 \frac{\ln x}{1+x}\,dx\\
    &=24 \int_0^1 \frac{\ln x}{1-x}\,dx-24\int_0^1 \frac{2x\ln x}{1-x^2}\,dx\\
    &=24 \int_0^1 \frac{\ln x}{1-x}\,dx-12\int_0^1 \frac{\ln x}{1-x}\,dx\\
    &=12 \int_0^1 \frac{\ln x}{1-x}\,dx\\
    &=-12\zeta(2)\\
    L&=\int_0^1 \ln^3(1-\sqrt{x})\,dx\\
    &\overset{y=1-\sqrt{x}}=2\int_0^1 (1-x)\ln^3 x\,dx\\
    &\overset{IPP}=-3\int_0^1 (2-x)\ln^2 x\,dx\\
    &\overset{IPP}=3\int_0^1 (4-x)\ln x\,dx\\
    &\overset{IPP}=-3\int_0^1 \left(4-\frac{x}{2}\right)\,dx\\
    &=-3\Big[4x-\frac{x^2}{4}\Big]_0^1\\
    &=-3\left(4-\frac{1}{4}\right)\\
    &=-\frac{45}{4}\\
    M&=\int_0^1 \ln^3(1-x)\,dx\\
    &\overset{y=1-x}=\int_0^1 \ln^3 x\,dx\\
    &\overset{IPP}=-3\int_0^1 \ln^2 x \,dx\\
    &\overset{IPP}=6\int_0^1 \ln x\,dx\\
    &\overset{IPP}=-6
    \end{align}
    Puisque pour $a,b$ réels, on a $(a-b)^3+(a+b)^3=2a^3+6ab$ alors on a aussi $2L+6J=K+M$
    Donc:\begin{align}
    J&=\frac{1}{6}\left(K+M-2L\right)\\
    &=\frac{1}{6}\left(-12\zeta(2)-6-2\times -\frac{45}{4}\right)\\
    &=-2\zeta(2)+\frac{11}{4}\\
    &=-2\times \frac{\pi^2}{6}+\frac{11}{4}\\
    &=\boxed{\frac{11}{4}-\frac{\pi^2}{3}}
    \end{align}
    J'ai utilisé:\begin{align}\int_0^1 \frac{\ln x}{1-x}\,dx&=-\zeta(2)\\
    \zeta(2)&=\frac{\pi^2}{6}
    \end{align}

    PS:
    On peut utiliser les mêmes trucs de Sioux (mathématiques) pour calculer l'intégrale qui ouvre l'autre fil de messages.
    L'automne est passé très vite cette année. B-)
  • Bonsoir AD, que la sentence tombe .
    J'ai jamais cru que je serais un jour lapidé par FDP, j'aimais sa compagnie
    Le 😄 Farceur


  • Gebrane:

    Ce n'est pas parce que tu tends un bâton pour te battre qu'on est obligé de l'utiliser contre toi.
  • Bonjour,

    @FDP : joli (tu)

    @gebrane : c'est toujours le même principe avec les logarithmes dans une intégrale entre $0$ et $1$.

    L'argument $\displaystyle (1+x)$ dans le logarithme est notre ennemi. Les arguments $\displaystyle (1-x), {1-x \over 1+x}, (1-x^2)$ sont nos amis.

    Pour se débarrasser de $\displaystyle \ln(1+x)$ on écrit simplement $\displaystyle \ln(1+x) =\ln (1-x) - \ln{1-x \over 1+x}$ ; pour se débarrasser de $\displaystyle \ln^2(1+x)$ on écrit $\displaystyle {1 \over 2} (\ln(1+x) + \ln(1-x))^2+{1 \over 2} (\ln(1+x) - \ln(1-x))^2 = \ln^2(1+x)+\ln^2(1-x).$ D'autres identités algébriques de puissance de $3$ ou $4$ ou encore plus grand servent à éliminer le $\displaystyle (1+x).$

    On transforme donc l'intégrale originale par une somme d'intégrales avec des arguments amis. Le calcul de ces intégrales amies est toujours le même : changement de variables $\displaystyle x \leadsto y$ où l'argument $\displaystyle (1-x), {1-x \over 1+x}, (1-x^2)$ devient $y$.

    Voilà ! Tu connais la recette. La prochaine fois essaie donc cette méthode.
  • L'argument (1+x) d'un log n'est pas toujours un ennemi, cela peut être un allié.

    Par exemple dans l'intégrale très connue

    $\displaystyle \int_0^1 \frac{\ln(1+x)}{1+x^2}\,dx$

    Ne pas oublier l'excellente propriété que par changement de variable $y=\dfrac{1-x}{1+x}$ , $\ln(1+x)$ se transforme en

    $\ln 2-\ln\left(1+y\right)$

    Cela permet pour le calcul de $\displaystyle \int_0^1\frac{\ln(1+x)\ln\left(\frac{1-x}{1+x}\right)\ln x}{1+x^2}\,dx$ de se ramener au calcul de $\displaystyle \int_0^1 \frac{\ln\left(\frac{1-x}{1+x}\right)\ln x}{1+x^2}\,dx$

    $\ln(1+x)$ est très moche a priori si on a en vue une utilisation de la fonction bêta d'Euler.
    Mais parfois on arrive à contourner le problème en utilisant $\ln(1+x)=\ln(1-x^2)-\ln(1-x)$

    PS:
    J'ai déterminé empiriquement une forme close pour la dernière intégrale c'est "moche".
    Il y a un terme en $\Im\left({\text{Li}_3\left(\frac{1+i}{2}\right)}\right)$ qui apparait.
  • Merci à vous FDP & Yves pour ces explications
    Au moins j'ai gardé ma tête sur les épaules dans cette expérience
    Merci AD:-)
    Le 😄 Farceur


  • FDP j’étais confiant lorsque j'ai mis ma tête aux enchères
    Je croyais que tu vas surement trouver la solution mais que tu va la garder pour toi au moins pour une semaine
    Pour YvesM ou P ou ... les plus expérimentés en calcul exact d'intégrales, on les voit se désintéresser à ce genre du fil
    Il restait donc les amateurs, c'est pour ce la que j’étais très confiant
    Mais ta solution très détaillée m'a surpris genre « Qu'on lui coupe la tête ! » :-D
    Le 😄 Farceur


  • Gebrane:

    Comme déjà indiqué par YvesM, avec un peu d'habitude, on sent ce qui va être faisable et quand j'ai vu ton intégrale il y a une petite voix dans ma tête qui m'a dit que ce truc était calculable et elle m'a indiquée une stratégie possible (un truc de Sioux mathématiques standard dans le domaine).
    Parfois, les premières impressions sont trompeuses mais comme tu as indiqué le résultat, et qu'il était relativement simple, cela a renforcé ma conviction que la stratégie entrevue allait surement fonctionner.

    Il ne faut pas croire que les séries type Euler avec des nombres harmoniques sont toujours la panacée pour calculer des intégrales.
    Si on connait une forme close d'une intégrale et qu'il y a terme type $\zeta(n)\zeta(m)$ avec $n,m$ deux entiers naturels non nuls différents cela peut plaider en faveur de l'utilisation de séries type séries d'Euler (on a des formes closes de telles séries où on trouve de tels produits).
    La présence d'un tel terme présage souvent un calcul difficile. B-)-
    Evidemment je mets de côté des intégrales comme $\displaystyle \int_0^1 \frac{\text{Li}_4(x)\text{Li}_2(x)}{x}\,dx$

    La formule $\displaystyle \int_0^1\int_0^1 xF(x)F(tx)\,dt\,dx=\frac{1}{2}\left(\int_0^1 F(x)\,dx\right)^2 $
    a un potentiel intéressant.

    PS: l'intégrale ci-dessus avec les polylogarithmes n'est pas simple à calculer mais par intégration par parties on sort un terme en $\zeta(3)\zeta(4)$ (ou $\zeta(5)\zeta(2)$ ). Mais on sait calculer des intégrales comme $\displaystyle \int_0^1 \frac{\text{Li}_{n+1}(x)\text{Li}_n(x)}{x}\,dx$ avec $n\geq 2$, un entier.
  • Celle ci ?
    $$\displaystyle\int_{0}^{10}\frac{\displaystyle (3+x^{\sqrt{7}})\sin(\frac{\pi}{2} x)dx}{6+x^{\sqrt{7}}+(10-x)^{\sqrt{7}}}=\frac{2}{\pi}.$$
    Le 😄 Farceur


  • Je vous propose, pour terminer l'année, de calculer:
    $\displaystyle \int_0^\infty \frac{\cos x\cos\left(\sqrt{1+x^2}\right)}{1+x^2}\,dx$
  • Quelques idées comme ça (je n'ai pas regardé en détails)
    on transforme cos(a)cos(b) en somme, puis changement de variable
    $\sqrt{1+x^2}+x=t$ dans une intégrale et $\sqrt{1+x^2}-x=t$ dans l'autre.
  • Bonjour,

    Je continue la méthode de @etanche :
    Les bornes changent de $1$ à l’infini et de $0$ à l’infini et donc de $0$ à l’infini par la relation de Chasles avec coupure en $1.$
    On tombe sur $\displaystyle \int_0^{+\infty} {2 \cos x\over 1+x^2}dx$ qui est classique.
    On aboutit à $\displaystyle I={\pi\over 2 e}.$ CORRECTION EFFECTUÉE.

    Un peu trop facile pour cette fin d’année.
  • Cette intégrale ne me semblait pas évidente à calculer. Mais Etanche a vu clair dans mon jeu.



    YvesM:
    Le résultat n'est pas exactement celui-là.


    En fait, je suis parti de l'intégrale, qu'YvesM a parfaitement identifiée et je me suis amusé à faire le calcul suivant que j'ai poussé aussi loin que je pouvais:
    \begin{align} \frac{\pi}{2\text{e}}&=\int_0^\infty \frac{\cos x}{1+x^2}\,dx\\
    &=\int_0^1 \frac{\cos x}{1+x^2}\,dx+\int_1^\infty\frac{\cos x}{1+x^2}\,dx\\
    &=\int_0^1 \frac{\cos x+\cos\left(\frac{1}{x}\right)}{1+x^2}\,dx\\
    &=2\int_0^1 \frac{\cos \left(\frac{1}{2}\left(x+\frac{1}{x}\right)\right)\cos \left(\frac{1}{2}\left(x-\frac{1}{x}\right)\right)}{1+x^2}\,dx\\
    &\overset{y=-\frac{1}{2}\left(x-\frac{1}{x}\right)}=\int_0^\infty \frac{\cos y\cos\left(\sqrt{1+y^2}\right)}{1+y^2}\,dy
    \end{align}

    Cela revient, dans l'autre sens, à faire quelque chose qui ressemble à ce qu'Etanche a décrit.
  • Je n'ai pas été assez malin.

    J'aurais dû plutôt adopter la présentation sous cette forme:
    $\displaystyle \int_0^{\frac{\pi}{2}} \cos\left(\frac{1}{\sin x}\right)\cos\left(\cot x\right)\,dx$
  • \begin{align}
    J&=\int_0^1\int_0^1\frac{\ln(1-xy)}{1-xy}\,dx\,dy\\
    &=\int_0^1 \frac{1}{x}\left(\int_0^1 \frac{x\ln(1-xy)}{1-xy}\,dy\right)\,dx\\
    &=\int_0^1 \frac{1}{x}\left(\int_0^x \frac{\ln(1-u)}{1-u}\,du\right)\,dx\\
    &=\int_0^1 \left(\frac{1}{x}\times -\frac{1}{2}\ln^2(1-x)\right)\,dx\\
    &=-\frac{1}{2}\int_0^1 \frac{\ln^2 x}{1-x}\,dx\\
    &=-\zeta(3)\\
    J&=\int_0^1 \frac{1}{x}\left(\int_0^x \frac{\ln(1-u)}{1-u}\,du\right)\,dx\\
    &=\left[\ln x\left(\int_0^x \frac{\ln(1-u)}{1-u}\,du\right)\right]_{x=0}^{x=1}-\int_0^1 \frac{\ln(1-x)\ln x}{1-x}\,dx\\
    &=-\int_0^1 \frac{\ln(1-x)\ln x}{1-x}\,dx\\
    \int_0^1 \frac{\ln(1-x)\ln x}{1-x}\,dx&=\zeta(3)

    \end{align} Et on retrouve la valeur d'une intégrale déjà bien connue en calculant $J$ de deux façons.
    Sachant que $\displaystyle \int_0^1 \frac{\ln^2 x}{1-x}\,dx=2\zeta(3)$

    PS:
    Je me rends compte que ce n'est pas d'un grand intérêt on peut faire une intégration par parties directement dans $\displaystyle \int_0^1 \frac{\ln(1-x)\ln x}{1-x}\,dx$
  • Je vous propose de montrer que: $\displaystyle \int_0^1 \frac{\ln(1-x)\ln^3 x}{1-x}\,dx=12\zeta(5)-6\zeta(2)\zeta(3)$

    Mais il y a un "cahier des charges" à respecter:
    1) Pas d'utilisation de sommes d'Euler, pas d'utilisation de séries à base de nombres harmoniques.
    2) Pas d'utilisation de la fonction Bêta d'Euler.
    Mais l'utilisation d'intégrales doubles est permise.


    NB: Sans l'utilisation d'intégrales doubles (et le théorème de Fubini) on voit mal comment faire apparaître le terme $\zeta(2)\zeta(3)$.
    La formule $0<a\leq 1, r\geq 1$, entier, $\displaystyle \int_0^1 \frac{\ln^r x }{1-ax}\,dx=\frac{(-1)^r r!}{a}\text{Li}_{r+1}(a)$
    est autorisée (et conseillée)

    PS:
    Je suis en train de finaliser le calcul demandé. Je suis à 90% sûr que ce que je vous propose est possible.
    (edit: j'ai terminé ce calcul. Ce que je propose est faisable, sauf erreur inattendue)

    PS2:
    $0\leq a\leq 1, r\geq 1$,$\displaystyle \text{Li}_{r}(a)=\sum_{n=1}^\infty \frac{a^n}{n^r}$
  • En me baladant sur la Toile j'ai trouvé ça :
  • Bonjour,

    @Chaurien :

    Pour $\displaystyle I=\int_0^1 {\ln (1+x) \over 1+x^2} dx$, après avoir montré l'existence facilement puisque l'intégrande est continu sur $[0,1]$, on effectue le CDV $x \leadsto \theta$ avec $\displaystyle x = \tan \theta$ et alors $\displaystyle I=\int_0^{\pi/4} d\theta \ln(1+\tan \theta)$ et comme $\displaystyle \cos \theta + \sin \theta = \sqrt{2} \sin (\theta + \pi/4)$ on trouve $\displaystyle I={\pi \over 8} \ln 2 + \int_0^{\pi/4} d\theta (\ln \sin(\theta + \pi/4) - ln \cos \theta) $ : l'intégrale est nulle par un CDV dans la première $\displaystyle \theta \leadsto t$ avec $\displaystyle t = \pi/4-\theta.$
  • J'ai déjà parlé de cette intégrale Ici

    C'est une injustice faite à Joseph Bertrand d'appeler cette intégrale: intégrale de Serret.
    Bertrand avait donné cette intégrale avec une méthode de calcul assez compliquée.
    Avant que Serret donne un calcul beaucoup plus simple.

    La vidéo, que j'ai parcourue rapidement, utilise une méthode trop compliquée à mon goût.

    Je préfère la méthode suggérée par YvesM ou:

    \begin{align}J&=\int_0^1 \frac{\ln(1+x)}{1+x^2}\,dx\\
    &\overset{y=\frac{1-x}{1+x}}=\int_0^1 \frac{\ln\left(\frac{2}{1+y}\right)}{1+y^2}\,dy\\
    &=\ln 2\int_0^1 \frac{1}{1+x^2}\,dx-J\\
    J&=\boxed{\dfrac{\pi}{8}\ln 2}
    \end{align}
  • Chaurien
    Modifié (07:28)
    Bravo à FdP, j'aime bien ces découvertes des origines des résultats mathématiques. Sérieuses, pas du « Al-Kashi ». La méthode de Bertrand a l'avantage de l'antériorité, et celle de Serret a l'avantage de la simplicité. On peut donc les réconcilier en adoptant l'appellation d'« intégrale de Bertrand-Serret », ce qui permet de la distinguer des intégrales de Bertrand bien connues. La méthode de la vidéo que j'ai trouvée me semble être celle de Bertrand, elle est à l'évidence fort laborieuse, et je lui préfère l'autre, de beaucoup. Mais il arrive souvent que l'on trouve d'abord un résultat par des voies compliquées, et que les suivants simplifient.

    Si l'on a une fonction $f$ définie et continue sur un segment $[a,b]$, $a<b$, dont le graphe admet pour centre de symétrie le point $(\frac {a+b}2, f(\frac {a+b}2))$, alors $\int_a^b {f(t)} dt=(b-a) f(\frac {a+b}2)$. Ceci se visualise géométriquement très simplement et se prouve immédiatement par un changement de variable affine très simple aussi. C'est le cas de la fonction $\theta \mapsto \ln(1+ \tan \theta)$ sur $[0, \frac {\pi}4]$. D'où ma préférence pour le changement de variable en $\tan \theta$, car le changement de variable $y=\frac {1-x}{1+x}$ me fait l'effet d'un lapin sortant d'un chapeau.

    Bonne après-midi.
    Fr. Ch.
  • Chaurien:

    Le changement de variable $y=\dfrac{1-x}{1+x}$ est la traduction dans le monde des "bêtes" fractions rationnelles du changement de variable trigonométrique $\displaystyle \theta=\tan\left(\frac{\pi}{4}-t\right)=\frac{1-\tan t}{1+\tan t}$
    C'est un changement de variable qui est souvent très utile même s'il est aussi souvent ignoré.

    Le calcul dont je parlais plus haut.

    \begin{align*}
    J&=\int_0^1 \frac{\ln(1-x)\ln^3 x}{1-x}\,dx,C=\int_0^1 \frac{\ln^3 t}{1-t}\,dt\\
    J&=\left[\left(\int_0^x \frac{\ln^3 t}{1-t}\,dt-C\right)\ln(1-x)\right]_0^1+\int_0^1 \frac{1}{1-x}\left(\int_0^x \frac{\ln^3 t}{1-t}\,dt-C\right)\,dx\\
    &=\int_0^1 \left(\left(\int_0^1 \frac{x\ln^3(tx)}{(1-x)(1-tx)}\,dt\right)-\frac{C}{1-x}\right)\,dx\\
    &=\int_0^1 \left(\left(\int_0^1 \frac{\ln^3(tx)}{(1-t)(1-x)}\,dt-\int_0^1 \frac{\ln^3(tx)}{(1-t)(1-tx)}\,dt\right)-\frac{C}{1-x}\right)\,dx\\
    &=6\left(\int_0^1 \frac{\ln^2 t}{1-t}\,dt\right)\left(\int_0^1 \frac{\ln x}{1-x}\,dx\right)+\\
    &\int_0^1 \left(\left(\int_0^1 \frac{\ln^3(x)}{(1-t)(1-x)}\,dt-\int_0^1 \frac{\ln^3(tx)}{(1-t)(1-tx)}\,dt\right)+\frac{C}{1-x}-\frac{C}{1-x}\right)\,dx\\
    &=6\left(\int_0^1 \frac{\ln^2 t}{1-t}\,dt\right)\left(\int_0^1 \frac{\ln x}{1-x}\,dx\right)+\int_0^1 \left(\int_0^1 \frac{\ln^3(x)}{(1-t)(1-x)}\,dt-\int_0^1 \frac{\ln^3(tx)}{(1-t)(1-tx)}\,dt\right)\,dx\\
    &=-12\zeta(2)\zeta(3)+\int_0^1 \left(\int_0^1 \frac{\ln^3(x)}{(1-t)(1-x)}\,dt-\int_0^1 \frac{\ln^3(tx)}{(1-t)(1-tx)}\,dt\right)\,dx\\
    &=-12\zeta(2)\zeta(3)+\int_0^1 \left(\frac{C}{1-t}-\frac{1}{t(1-t)}\left(\int_0^t \frac{\ln^3 u}{1-u}\,du\right)\right)\,dt\\
    0&\leq A<1\\
    J(A)&=-12\zeta(2)\zeta(3)+\int_0^A \left(\frac{C}{1-t}-\frac{1}{t(1-t)}\left(\int_0^t \frac{\ln^3 u}{1-u}\,du\right)\right)\,dt\\
    &=-12\zeta(2)\zeta(3)-C\ln(1-A)-\int_0^A \frac{1}{t(1-t)}\left(\int_0^t \frac{\ln^3 u}{1-u}\,du\right)\,dt\\
    &=-12\zeta(2)\zeta(3)-C\ln(1-A)-\left[\ln\left(\frac{t}{1-t}\right)\left(\int_0^t \frac{\ln^3 u}{1-u}\,du\right)\right]_0^A+\int_0^A \frac{\ln\left(\frac{t}{1-t}\right)\ln^3 t}{1-t}\,dt\\
    &=-12\zeta(2)\zeta(3)+\ln(1-A)\left(\left(\int_0^A \frac{\ln^3 u}{1-u}\,du\right)-C\right)-\ln(A)\left(\int_0^A \frac{\ln^3 u}{1-u}\,du\right)+\\
    &\int_0^A \frac{\ln^4 t}{1-t}\,dt-\int_0^A \frac{\ln(1-t)\ln^3 t}{1-t}\,dt\\
    J&=\lim_{A\rightarrow 1}J(A)\\
    &=-12\zeta(2)\zeta(3)+\int_0^1 \frac{\ln^4 t}{1-t}\,dt-J\\
    &=-12\zeta(2)\zeta(3)+24\zeta(5)-J\\
    J&=\boxed{12\zeta(5)-6\zeta(2)\zeta(3)}
    \end{align*}
  • Bravo pour ce calcul impressionnant (tu).
    Pour en revenir à l'exemple moins ambitieux que j'ai évoqué plus haut, on n'est pas obligé de se contenter de $ \displaystyle \int_{0}^{1}\frac{\ln (1+t)}{1+t^{2}}dt$, et l'on pourrait calculer aussi $\displaystyle \int_{0}^{+\infty }\frac{\ln (1+t)}{1+t^{2}}dt$. La valeur de cette dernière intégrale fait intervenir la constante de Catalan $\displaystyle G=\overset{+\infty }{\underset{n=0}{\sum }}\frac{(-1)^{n}}{(2n+1)^{2}}\simeq 0,91596559$.
    Cette constante a été présentée par Eugène Catalan dans une communication à l'Académie des Sciences en 1864, avec un grand nombre de calculs d'intégrales.
    https://books.google.fr/books?id=LXZFAAAAcAAJ&pg=PA618&redir_esc=y#v=onepage&q&f=false
    On peut aussi calculer $ \displaystyle \int_{0}^{1}\frac{\ln (1+t^2)}{1+t^{2}}dt$ et $\displaystyle \int_{0}^{+\infty }\frac{\ln (1+t^2)}{1+t^{2}}dt$, certainement déjà connues de FdP.
    À côté de ses incroyables intégrales, c'est de la roupie de sansonnet, mais moi ça m'intéresse, et ça peut être posé en colles dans les classes où c'est au programme, ou à l'Université.
    Bonne journée. On gagne quelques minutes de soleil chaque jour, j'aime.
    Fr. Ch.
  • Chaurien:

    Dans ce fil il y a un lien sur le mémoire dont un extrait est cité dans le document que tu as mis en lien.

    Toutes les intégrales que tu donnes me sont connues. La routine. B-)-
  • J'admire encore plus FdP d'avoir intuité que $S=\frac{1}{6}\int_0^1\frac{\log(1-x)}{1-x}\log ^3x\, \frac{dx}{x}=2\zeta(5)-\zeta(2)\zeta(3)$ que de l'avoir démontré par un long calcul. Montrons ce résultat d'une autre manière en observant que $$S=\frac{1}{6}\int_0^{\infty}\frac{\log(1-e^{-u})}{1-e^{-u}}u^3du=\frac{1}{6}\sum_{n=1}^{\infty}\sum_{k=0}^{\infty}\frac{1}{n}\int_{0}^{\infty}e^{-(n+k)u}u^3du=\sum_{n=1}^{\infty}\sum_{k=0}^{\infty}\frac{1}{n(n+k)^4}=\sum_{m=1}^{\infty}\sum_{n=1}^{m}\frac{1}{nm^4}.$$ On se débarrasse d'un premier $\zeta(5)$ avec $S= \zeta(5)+S_1$ avec $$S_1=\sum_{m=1}^{\infty}\sum_{n=1}^{m-1}\frac{1}{nm^4}=\sum_{m=1}^{\infty}\sum_{n=1}^{m-1}\frac{1}{(m-n)m^4}.$$ Ensuite on utilise la décomposition en éléments simples $$
    m\mapsto \frac{1}{(m-n)m^4}=\frac{1}{(m-n)n^4}-\frac{1}{mn^4}-\frac{1}{m^2n^3}-\frac{1}{m^3n^2}$$ pour écrire $S_1=S_2-S_3,$ avec $$
    S_2= \sum_{m=1}^{\infty}\sum_{n=1}^{m-1}\Big(\frac{1}{(m-n)n^4}-\frac{1}{mn^4}\Big)=0,\ S_3=\sum_{m=1}^{\infty}\sum_{n=1}^{m-1}\left(\frac{1}{m^2n^3}+\frac{1}{m^3n^2}\right)=-\zeta(5)+\zeta(2)\zeta(3).
    $$ J'ai la flemme d'appliquer cette méthode à $\displaystyle \int_0^1\frac{\log(1-x)}{1-x}\log ^{2p-1}x \, \frac{dx}{x}.$
  • P:
    Je ne l'ai pas réellement deviné. Je savais qu'on pouvait l'exprimer comme une combinaison linéaire rationnelles des nombres $\zeta(5),\zeta(2)\zeta(3)$. Pour trouver les coefficients c'est facile avec PARI GP.

    Les intégrales du type $\displaystyle \int_0^1 \frac{\ln(1-x)\ln^n x}{1-x}\,dx$ se traitent par des séries harmoniques.

    \begin{align} \int_0^1 \frac{\ln(1-x)\ln^s x}{1-x}\,dx=-\int_0^1 \left(\sum_{n=0}^\infty \text{H}_n x^n\right)\ln^s x\,dx=\text{constante}\sum_{n=1}^\infty \frac{\text{H}_n}{n^{s+1}}\end{align}

    Et il y a une formule connue depuis Euler qui permet de donner une forme close à cette série si je me souviens bien.*

    Autrement on peut considérer la fonction Bêta $B(s,t)=\int_0^1 x^s(1-x)^t\,dx$ et dériver en fonction de $t,s$.
    (fonction pour laquelle on a une formule en fonction de valeurs de la fonction Gamma)

    Pour certains cas, on peut procéder comme tu l'as fait.

    *: on trouve cette formule à la page 6 de ce papier: http://algo.inria.fr/flajolet/Publications/FlSa98.pdf
  • Merci FdP pour cette belle reference a Philippe Flajolet.
  • Mathématicien parti trop tôt.
  • Bonjour
    @Fin de partie dans ton calcul de l'intégrale de J, je ne comprends pas ton passage de la quatrième égalité à la cinquième. L'apparition du facteur 6 fois les 2 intégrales , et surtout l'ajout du terme C/(1-x) dans l'intégrale de la quatrième égalité. En effet $\int_0^1 C/(1-x) dx$ est divergente. Y-a-t-il quelque chose qui m'échappe où est-ce une erreur? Merci d'avance si tu pouvais expliquer un peu ce passage.
     
  • BD2017:

    \begin{align}S&=\int_0^1 \left(\left(\int_0^1\frac{\ln^3(tx)}{(1-t)(1-x)}\,dt-\int_0^1 \frac{\ln^3(tx)}{(1-t)(1-tx)}\,dt\right)-\frac{C}{1-x}\right)\,dx\\
    &=\int_0^1 \left(\int_0^1\left(\frac{\ln^3 t}{(1-t)(1-x)}\,dt+\frac{\ln^3 x}{(1-t)(1-x)}+\frac{3\ln^2 t\ln x}{(1-t)(1-x)}+\frac{3\ln t\ln^2 x}{(1-t)(1-x)}-\int_0^1 \frac{\ln^3(tx)}{(1-t)(1-tx)}\right)dt-\frac{C}{1-x}\right)dx\end{align}

    Les termes dans lesquels figurent au numérateur un produit de $\ln x$ par $\ln t$ donnent des intégrales convergentes et peuvent être sortis, ces termes donnent des fonctions qui sont convergentes quand on les intègre sur le carré $[0;1]\times [0,1]$.
    \begin{align}S&=\int_0^1 \left(\int_0^1\left(\frac{\ln^3 t}{(1-t)(1-x)}+\frac{\ln^3 x}{(1-t)(1-x)}-\int_0^1 \frac{\ln^3(tx)}{(1-t)(1-tx)}\right)dt-\frac{C}{1-x}\right)dx+\\
    &3\int_0^1\int_0^1\frac{\ln^2 t\ln x}{(1-t)(1-x)}\,dt\,dx+3\int_0^1 \int_0^1\frac{\ln t\ln^2 x}{(1-t)(1-x)}\,dt\,dx\end{align}
    Les deux derniers termes sont égaux par symétrie et leur somme vaut, en appliquant un théorème type Fubini:
    $\displaystyle 6\left(\int_0^1\frac{\ln t}{1-t}\,dt\right)\left(\int_0^1\frac{\ln^2 x}{1-x}\,dx\right)$


    \begin{align}T&=\int_0^1 \left(\int_0^1\left(\frac{\ln^3 t}{(1-t)(1-x)}+\frac{\ln^3 x}{(1-t)(1-x)}-\int_0^1 \frac{\ln^3(tx)}{(1-t)(1-tx)}\right)dt-\frac{C}{1-x}\right)dx\\
    &=\int_0^1 \left(\int_0^1\left(\frac{\ln^3 x}{(1-t)(1-x)}-\int_0^1 \frac{\ln^3(tx)}{(1-t)(1-tx)}\right)dt+\frac{C}{1-x}-\frac{C}{1-x}\right)dx\\&=\int_0^1 \left(\int_0^1\left(\frac{\ln^3 x}{(1-t)(1-x)}-\int_0^1 \frac{\ln^3(tx)}{(1-t)(1-tx)}\right)dt\right)dx\\
    &=\int_0^1 \left(\int_0^1\left(\frac{\ln^3 x}{(1-t)(1-x)}-\int_0^1 \frac{\ln^3(tx)}{(1-t)(1-tx)}\right)dx\right)dt\\
    &\int_0^1 \left(\frac{C}{1-t}-\left(\int_0^1 \frac{\ln^3(tx)}{(1-t)(1-tx)}\,dx\right)\right)\,dt
    \end{align}

    Et après, on fait le changement de variable $u(x)=tx$ dans l'intégrale intérieure.



    PS:
    Je pense que j'ai corrigé ce qu'il fallait.

    PS2:
    Ce calcul peut surement être généralisé pour donner une "nouvelle" preuve de la formule d'Euler mentionnée plus haut.
  • Merci beaucoup @FDP. Je comprends.
    Néanmoins c'est assez recherché donc bravo.
     
  • Quand tu vois la forme du résultat (la présence de $\zeta(2)\zeta(3)$ cela suggère que va intervenir :

    $\displaystyle \left(\int_0^1\frac{\ln t}{1-t}\,dt\right)\left(\int_0^1\frac{\ln^2 x}{1-x}\,dx\right)$

    C'est presque du reverse-engineering.

    En tout cas, c'est ce qui m'a rendu confiant sur la faisabilité de ce que j'ai exposé plus haut.
  • Bonsoir.
    Je relis ce fil de discussion et je vois il y a quatre semaines : $ \int_0^{+ \infty} \frac{\cos x}{1+x^2}\,dx=\frac {\pi}{2e}$. Malgré mon peu de compétence en analyse complexe, je crois savoir qu'on démontre généralement cette égalité avec une intégration sur un contour constitué d'un demi-cercle et de son diamètre, et le théorème des résidus. On obtient aussi de même : $ \int_0^{+ \infty} \frac{x \sin x}{1+x^2}\,dx=\frac {\pi}{2e}$. Quelqu'un connaît-il un autre procédé de calcul de ces intégrales ?
    Autre question : en fait la méthode que j'ai évoquée donne : $ \int_{- \infty}^{+ \infty} \frac{\cos x}{1+x^2}\,dx=\frac {\pi}{e}$ et $ \int_{- \infty}^{+ \infty} \frac{ x \sin x}{1+x^2}\,dx=\frac {\pi}{e}$, et puis division par $2$. Bien sûr on ne peut utiliser cette méthode pour calculer $ \int_0^{+ \infty} \frac{\sin x}{1+x^2}\,dx$. Y a-t-il moyen de calculer cette dernière ?
    Bonne fin d'après-midi.
    Fr. Ch.
  • D'apres 3.766 1. de Gradshteyn et Ryzhik, ce n'est pas elementaire.
  • Cela dépend de ce qu'on appelle élémentaire.
    Wolfy donne une forme close:
    https://www.wolframalpha.com/input/?i=integrate+sin(x)/(1+x^2),x=0,infinity

    (mais il faut parler d'exponentielle intégrale)
  • Bonjour,

    @Chaurien : L'intégrale $\displaystyle \int_{0}^{+\infty} {\sin x \over 1+x^2} dx$ se calcule bien, mais il est bon de connaître la fonction exponentielle intégrale (ce qui ne vient pas tout seul).

    Pour tout $\displaystyle z \in \C^*$, on a $\displaystyle Ei(z) = \sum_{k \geq 1} {z^k \over k.k!} + \gamma + {1\over 2} (\ln z - \ln{1 \over z}).$

    On montre alors que, pour tout $\displaystyle z \in \C^*, {d \over dz} Ei(z) ={e^z \over z}.$

    On montre l'existence de l'intégrale puisque l'intégrande est continu sur $\displaystyle [0, +\infty[$ et $\displaystyle |\int_{0}^{+\infty} {e^{i x} \over 1+x^2} dx| \leq \int_{0}^{+\infty} {1 \over 1+x^2} dx = {\pi \over 2}.$

    Il suffit de décomposer en fraction simples : $\displaystyle {1 \over 1+x^2} = {1 \over 2i} ({1\over x-i} -{1\over x+i})$ et alors, immédiatement, on calcule, pour tout $x \in \R$, $\displaystyle {d \over dx} Ei(1+i x) =i {e^{1+ix} \over 1+ix}$ et $\displaystyle {d \over dx} Ei(-1+i x) =i {e^{-1+ix} \over -1+ix}.$
    On a donc établi que, pour tout $x \in \R$, $\displaystyle {d \over dx} {1 \over 2i} ({1 \over e}Ei(1+i x) -e Ei(-1+i x) ) = {e^{ix} \over 1+x^2}.$

    L'intégrale est alors $\displaystyle \int_{0}^{+\infty} {e^{i x} \over 1+x^2} dx= {1 \over 2i} ({1 \over e}Ei(1+i x) -e Ei(-1+i x) )|_{0}^{+\infty} .$

    Ici, il faut faire attention aux limites car la fonction exponentielle intégrale possède une coupure. La limite en zéro est par valeurs supérieures (positives). La dernière limite ci-dessous est la difficulté du calcul.

    On a :
    $\displaystyle Ei(1+ix) \to i \pi, (x \to +\infty)$,
    $\displaystyle Ei(1+ix) \to Ei(1), (x \to 0^+)$,
    $\displaystyle Ei(-1+ix) \to i \pi, (x \to +\infty)$,
    $\displaystyle Ei(-1+ix) \to Ei(-1)+i \pi, (x \to 0^+).$

    Et alors, on trouve : $\displaystyle \int_{0}^{+\infty} {e^{i x} \over 1+x^2} dx= {1 \over 2i} ({1 \over e}(i \pi - Ei(1)) -e (i \pi - (Ei(-1) + i \pi)) ) ={\pi \over 2} {1 \over e} + {i \over 2} ({1 \over e}Ei(1) - e Ei(-1)).$

    On a donc montré que $\displaystyle \int_{0}^{+\infty} {\cos x \over 1+x^2} dx= {\pi \over 2} {1 \over e} $ et $\displaystyle \int_{0}^{+\infty} {\sin x \over 1+x^2} dx= {1 \over 2} ({1 \over e}Ei(1) - e Ei(-1)).$
  • Merci pour cette explication détaillée. Effectivement, tout dépend de la panoplie de fonctions réputées élémentaires que l'on s'autorise à utiliser.
    Bonne soirée.
    Fr. Ch.
  • Bonjour,

    Voici comment je montre que $\displaystyle \int_0^{+\infty} dx {\cos x\over 1+x^2}=\int_0^{+\infty} dx {x\sin x\over 1+x^2}.$

    Pour tout $a\geq 0$, on définit $\displaystyle F(a)=\int_0^{+\infty} dx {\cos x\over 1+x^2} e^{-a x}.$

    On établit sans peine $\displaystyle \int_0^{+\infty} dx \cos x e^{-a x}={a\over 1+a^2}$ puis l’équation différentielle $F”(a)+F(a)=f(a)={a\over 1+a^2}.$

    Les solutions sont données par $\displaystyle F(a)=A \cos a+B\sin a+C(a) \sin a -S(a) \cos a$ avec $A,B$ deux constantes et $\displaystyle C(a)=\int_0^a dx f(x) \cos x, S(a)=\int_0^a dx f(x) \sin x.$

    Si on ne le sait pas, on le vérifie par substitution.

    On observe que $\displaystyle F(a)\to 0,(a\to +\infty).$ On en déduit $\displaystyle F(a)=\int_a^{+\infty} dx {x \sin (x-a)\over 1+x^2}=\int_0^{+\infty} dx {\cos x\over 1+x^2} e^{-a x}.$

    La relation cherchée est $F(0).$
  • Je cherchais une manière élémentaire de donner une forme close (qui est bien connue) à $\displaystyle \text{Li}_3\left(\frac{1}{2}\right)$.

    \begin{align}
    K&=\int_0^{\frac{1}{2}}\left(\frac{\ln^2\left(\frac{x}{1-x}\right)}{x}-\frac{\ln^2 x}{x}\right)\,dx\\
    &=\int_0^{\frac{1}{2}} \left(\frac{\ln^2(1-x)}{x}-\frac{2\ln x\ln(1-x)}{x}\right)\,dx\\
    &=\int_0^{\frac{1}{2}} \frac{\ln^2(1-x)}{x}\,dx-\Big[\ln^2 x\ln(1-x)\Big]_0^{\frac{1}{2}}-\int_0^{\frac{1}{2}} \frac{\ln^2 x}{1-x}\,dx\\
    &=\int_0^{\frac{1}{2}} \frac{\ln^2(1-t)}{t}\,dt+\ln^3 2-\int_0^{\frac{1}{2}} \frac{\ln^2 x}{1-x}\,dx\\
    &\overset{x=1-t}=\int_{\frac{1}{2}}^1 \frac{\ln^2 x}{1-x}\,dx+\ln^3 2-\int_0^{\frac{1}{2}} \frac{\ln^2 x}{1-x}\,dx\\
    &=\int_0^1 \frac{\ln^2 x}{1-x}\,dx+\ln^3 2-2\int_0^{\frac{1}{2}} \frac{\ln^2 t}{1-t}\,dt\\
    &\overset{x=2t}=2\zeta(3)+\ln^3 2-\int_0^{1} \frac{\ln^2\left(\frac{x}{2}\right)}{1-\frac{x}{2}}\,dx\\
    &=2\zeta(3)+\ln^3 2-\int_0^{1} \frac{\ln^2 x}{1-\frac{x}{2}}\,dx-\ln^2 2\int_0^{1} \frac{1}{1-\frac{x}{2}}\,dx+2\ln 2\int_0^1 \frac{\ln x}{1-\frac{x}{2}}\,dx\\
    &=2\zeta(3)+\ln^3 2-4\text{Li}_3\left(\frac{1}{2}\right)-\ln^2 2\int_0^{1} \frac{1}{1-\frac{x}{2}}\,dx+2\ln 2\int_0^1 \frac{\ln x}{1-\frac{x}{2}}\,dx\\
    &=2\zeta(3)-\ln^3 2-4\text{Li}_3\left(\frac{1}{2}\right)+2\ln 2\int_0^1 \frac{\ln x}{1-\frac{x}{2}}\,dx\\
    \end{align}

    Par ailleurs,

    \begin{align}
    K&\overset{y=\frac{x}{1-x}}=\int_0^1 \left(\frac{\ln^2 x}{x(1+x)}-\frac{\ln^2\left(\frac{x}{1+x}\right)}{x(1+x)}\right)\,dx\\
    &=\int_0^1 \left(\frac{2\ln x\ln(1+x)}{x(x+1)}-\frac{\ln^2(1+x)}{x(x+1)}\right)\,dx\\
    &=2\int_0^1 \frac{\ln x\ln(1+x)}{x}\,dx-2\int_0^1 \frac{\ln x\ln(1+x)}{1+x}\,dx-\\
    &\int_0^1 \frac{\ln^2(1+x)}{x}\,dx+\frac{1}{3}\ln^3 2\\
    &=\Big(\Big[\ln^2 x\ln(1+x)\Big]_0^1-\int_0^1 \frac{\ln^2 x}{1+x}\,dx\Big)-2\int_0^1 \frac{\ln x\ln(1+x)}{1+x}\,dx-\\
    &\Big(\Big[\ln x\ln^2(1+x)\Big]_0^1-2\int_0^1 \frac{\ln x\ln(1+x)}{1+x}\,dx\Big)+\frac{1}{3}\ln^3 2\\
    &=-\int_0^1 \frac{\ln^2 x}{1+x}\,dx+\frac{1}{3}\ln^3 2\\
    &=-\int_0^1 \frac{\ln^2 x}{1-x}\,dx+\int_0^1 \frac{2t\ln^2 t}{1-t^2}\,dt+\frac{1}{3}\ln^3 2\\
    &\overset{x=t^2}=-\int_0^1 \frac{\ln^2 x}{1-x}\,dx+\frac{1}{4}\int_0^1 \frac{\ln^2 x}{1-x}\,dt+\frac{1}{3}\ln^3 2\\
    &=\frac{1}{3}\ln^3 2-\frac{3}{4}\int_0^1 \frac{\ln^2 x}{1-x}\,dx\\
    &=\frac{1}{3}\ln^3 2-\frac{3}{2}\zeta(3)\\
    \end{align}
    Donc,
    \begin{align}
    -4\text{Li}_3\left(\frac{1}{2}\right)&=\frac{4}{3}\ln^3 2-\frac{7}{2}\zeta(3)-2\ln 2\int_0^1 \frac{\ln x}{1-\frac{x}{2}}\,dx\\
    \int_0^1 \frac{\ln t}{1-\frac{t}{2}}\,dt&\overset{x=\frac{t}{2-t}}=2\int_0^1 \frac{\ln\left(\frac{2x}{1+x}\right)}{1+x}\,dx\\
    &=2\int_0^1 \frac{\ln x}{1+x}\,dx-\left[\ln^2\left(\frac{2}{1+x}\right)\right]_0^1\\
    &=2\int_0^1 \frac{\ln x}{1+x}\,dx+\ln^2 2\\
    &=2\int_0^1 \frac{\ln x}{1-x}\,dx-2\int_0^1 \frac{2t\ln t}{1-t^2}\,dt+\ln^2 2\\
    &\overset{x=t^2}=2\int_0^1 \frac{\ln x}{1-x}\,dx-\int_0^1 \frac{\ln x}{1-x}\,dx+\ln^2 2\\
    &=\int_0^1 \frac{\ln x}{1-x}\,dx+\ln^2 2\\
    &=\ln^2 2-\zeta(2)
    \end{align}
    Ainsi,
    \begin{align}
    \text{Li}_3\left(\frac{1}{2}\right)&=\frac{1}{6}\ln^3 2+\frac{7}{8}\zeta(3)-\frac{1}{2}\zeta(2)\ln 2\\
    &=\boxed{\frac{1}{6}\ln^3 2+\frac{7}{8}\zeta(3)-\frac{1}{12}\pi^2\ln 2}
    \end{align}
    NB:
    J'utilise: Pour $r\geq 1,0< a\leq 1,\displaystyle \int_0^1 \frac{\ln^r x }{1-ax}\,dx=\frac{(-1)^r r!}{a}\text{Li}_{r+1}(a)$
  • Si on prend plutôt,

    $\displaystyle K=\int_0^{\frac{1}{2}}\left(\frac{\ln^2\left(\frac{x}{1-x}\right)}{x(1-x)}-\frac{\ln^2 x}{x}\right)\,dx$

    je pense que cela permet de raccourcir un peu le calcul précédent.
Connectez-vous ou Inscrivez-vous pour répondre.